Pregunta # 82567

Pregunta # 82567
Anonim

Resposta:

#cos ((2pi) / 9) + isin ((2pi) / 9) #, #cos ((8pi) / 9) + isin ((8pi) / 9) # i

#cos ((14pi) / 9) + isin ((14pi) / 9) #,

Explicació:

El primer que cal fer és posar el número en forma de # rhoe ^ (thetai) #

# rho = sqrt ((1/2) ^ 2 + (sqrt (3) / 2) ^ 2) = sqrt (1/4 + 3/4) = 1

# theta = arctan ((sqrt (3) / 2) / (- 1/2)) = arctan (-sqrt (3)) = - pi / 3 + kpi #. Escollim # (2pi) / 3 #ja que estem en el segon quadrant. Tingueu en compte això # -pi / 3 # està en el quart quadrant, i això està malament.

El vostre número és ara:

# 1e ^ ((2pii) / 3) #

Ara les arrels són:

#root (3) (1) e ^ (((2kpi + (2pi) / 3) i) / 3), k en ZZ #

# = e ^ ((((6kpi + 2pi) i) / 9), k en ZZ #

per tant, podeu triar k = 0, 1, 2 i obtenir:

#e ^ ((2pii) / 9 #, #e ^ ((8kpii) / 9 # i #e ^ ((14kpii) / 9 #

o bé #cos ((2pi) / 9) + isin ((2pi) / 9) #, #cos ((8pi) / 9) + isin ((8pi) / 9) # i

#cos ((14pi) / 9) + isin ((14pi) / 9) #.

Per a mi, aquest és un punt mort, perquè no puc calcular funcions trigonomètriques de múltiples de # pi / 9 #. Hem de confiar en una calculadora:

# 0.7660 + 0.6428i #

# -0.9397 + 0.3420i #

# 0.1736-0.9848i #